Last visit was: 25 Apr 2024, 00:20 It is currently 25 Apr 2024, 00:20

Close
GMAT Club Daily Prep
Thank you for using the timer - this advanced tool can estimate your performance and suggest more practice questions. We have subscribed you to Daily Prep Questions via email.

Customized
for You

we will pick new questions that match your level based on your Timer History

Track
Your Progress

every week, we’ll send you an estimated GMAT score based on your performance

Practice
Pays

we will pick new questions that match your level based on your Timer History
Not interested in getting valuable practice questions and articles delivered to your email? No problem, unsubscribe here.
Close
Request Expert Reply
Confirm Cancel
SORT BY:
Kudos
Tags:
Show Tags
Hide Tags
Intern
Intern
Joined: 04 May 2015
Posts: 2
Own Kudos [?]: 1 [1]
Given Kudos: 3
Send PM
Tutor
Joined: 16 Oct 2010
Posts: 14820
Own Kudos [?]: 64906 [1]
Given Kudos: 426
Location: Pune, India
Send PM
CEO
CEO
Joined: 27 Mar 2010
Posts: 3675
Own Kudos [?]: 3528 [1]
Given Kudos: 149
Location: India
Schools: ISB
GPA: 3.31
Send PM
GMAT Club Verbal Expert
Joined: 13 Aug 2009
Status: GMAT/GRE/LSAT tutors
Posts: 6920
Own Kudos [?]: 63659 [1]
Given Kudos: 1773
Location: United States (CO)
GMAT 1: 780 Q51 V46
GMAT 2: 800 Q51 V51
GRE 1: Q170 V170

GRE 2: Q170 V170
Send PM
Re: Aroca City currently funds its public schools through taxes on [#permalink]
1
Kudos
Expert Reply
kashishDhir wrote:
perezhan wrote:
Aroca City currently funds its public schools through taxes on property. In place of this system, the city plans to introduce a sales tax of three percent on all retail sales in the city. Critics protest that three percent of current retail sales falls short of the amount raised for schools by property taxes. The critics are correct on this point. Nevertheless, implementing the plan will probably not reduce the money going to Aroca's schools. Several large retailers have selected Aroca City as the site for huge new stores, and these are certain to draw large numbers of shoppers from neighboring municipalities, where sales are taxed at rates of six percent and more. In consequence, retail sales in Aroca City are bound to increase substantially.

In the argument given, the two potions in boldface play which of the following roles?

(A) The first is an objection that has been raised against a certain plan; the second is a prediction that, if accurate, undermines the force of that objection.

(B) The first is a criticism, endorsed by the argument, of a funding plan; the second is a point the argument makes in favor of adopting an alternative plan.

(C) The first is a criticism, endorsed by the argument, of a funding plan; the second is the main reason cited by the argument for its endorsement of the criticism.

(D) The first is a claim that the argument seeks to refute; the second is the main point used by the argument to show that the claim is false.

(E) The first is a claim that the argument accepts with certain reservations; the second presents that claim in a rewarding that is not subject to those reservations.


Similar but not the same question from OG is here:
https://gmatclub.com/forum/aroca-city-c ... 37810.html

VeritasKarishma wrote:
Hoozan wrote:
GMATNinja VeritasKarishma EducationAisle


Can you help me bifurcate each sentence into premise, conclusion etc.
Aroca City currently funds its public schools through taxes on property. In place of this system, the city plans to introduce a sales tax of three percent on all retail sales in the city.-Premise
Critics protest that three percent of current retail sales falls short of the amount raised for schools by property taxes. -Premise
The critics are correct on this point.- Intermediate conclusion as per author
Nevertheless, implementing the plan will probably not reduce the money going to Aroca's schools. - Counter premise
Several large retailers have selected Aroca City as the site for huge new stores, and these are certain to draw large numbers of shoppers from neighboring municipalities, where sales are taxed at rates of six percent and more.-Counter premise
In consequence, retail sales in Aroca City are bound to increase substantially- Main conclusion

When you're breaking down a passage, you want to think about the most important thing first. Then piece together how the author builds up to that important thing. Why did the author write the passage in the first place?

Here, the author is trying to convince the reader that "implementing the plan [of replacing property tax with sales tax] will probably not reduce the money going to Aroca's schools." THIS is the most important piece of the passage -- it is the author's conclusion. Everything else in the passage revolves around this conclusion as support, or background context, or a counterargument, etc.

So, how does the author support this conclusion? By building the following chain of logic:
  • First, the author lists this piece of supporting evidence: "Several large retailers have selected Aroca City as the site for huge new stores, and these are certain to draw large numbers of shoppers from neighboring municipalities, where sales are taxed at rates of six percent and more."
  • Then, the author makes a prediction based on the above fact: "In consequence, retail sales in Aroca City are bound to increase substantially."

Based on these two pieces of support, the author concludes that Aroca's school won't get less money under the new plan.

So, what else is in the passage?

The first two sentences just give us background information and context for the rest of the argument: "Aroca City currently funds its public schools through taxes on property. In place of this system, the city plans to introduce a sales tax of three percent on all retail sales in the city."

Then, the author give us the perspective of some critics of the plan: "Critics protest that three percent of current retail sales falls short of the amount raised for schools by property taxes."

The author actually agrees with this point. However, he/she then goes on to say why the plan isn't such a bad idea, even if the critics are right that taxing CURRENT retail sales won't provide as much money as property taxes do.

Overall, this is how the argument is constructed:
    1) Background info: "Aroca City currently funds its public schools through taxes on property. In place of this system, the city plans to introduce a sales tax of three percent on all retail sales in the city."
    2) Critic's viewpoint: "Critics protest that three percent of current retail sales falls short of the amount raised for schools by property taxes."
    3) The author saying that the critics are right: "The critics are correct on this point."
    4) The author's conclusion, which he/she reaches in spite of the above objection: "Nevertheless, implementing the plan will probably not reduce the money going to Aroca's schools."
    5) The author's support for this conclusion: "Several large retailers have selected Aroca City as the site for huge new stores, and these are certain to draw large numbers of shoppers from neighboring municipalities, where sales are taxed at rates of six percent and more. In consequence, retail sales in Aroca City are bound to increase substantially."

I hope that helps!
Director
Director
Joined: 20 Apr 2022
Posts: 628
Own Kudos [?]: 254 [1]
Given Kudos: 315
Location: India
GPA: 3.64
Send PM
Re: Aroca City currently funds its public schools through taxes on [#permalink]
1
Kudos
avigutman I do not get it how the first BF is not being refuted. TBH if 3% of retail sales will be reduction in the amount of funds raised for school, and there will be no reduction in the money given to schools due to higher retail sales then how can we say 3% will be a reduction? If retail sales are increasing, it is rather likely that the critics' claim (3% of retail sales being less) is wrong because with increased retail sales, 3% will be more
Tutor
Joined: 17 Jul 2019
Posts: 1304
Own Kudos [?]: 2286 [1]
Given Kudos: 66
Location: Canada
GMAT 1: 780 Q51 V45
GMAT 2: 780 Q50 V47
GMAT 3: 770 Q50 V45
Send PM
Aroca City currently funds its public schools through taxes on [#permalink]
1
Kudos
Expert Reply
Elite097 wrote:
avigutman I do not get it how the first BF is not being refuted. TBH if 3% of retail sales will be reduction in the amount of funds raised for school, and there will be no reduction in the money given to schools due to higher retail sales then how can we say 3% will be a reduction? If retail sales are increasing, it is rather likely that the critics' claim (3% of retail sales being less) is wrong because with increased retail sales, 3% will be more


Elite097 The sentence "The critics are correct on this point." shows that the argument does not seek to refute the first BF.
You misrepresented the claim made by the first BF in your quote above (see the verb tense you used, which I boldfaced).
The claim was
Quote:
three percent of current retail sales falls short of the amount raised for schools by property taxes

Note the parts that I boldfaced... This claim is a claim about the current state of the world, not about the future.

Edited to add: in my opinion, boldface CR questions are most easily, accurately, and quickly solved by NOT reading the actual boldfaced text.
I know this is extremely counterintuitive, but that's what I do and that's what I teach.
In this particular problem, I only read the following highlights:
Quote:
Aroca City currently funds its public schools through taxes on property. In place of this system, the city plans to introduce a sales tax of three percent on all retail sales in the city. Critics protest that three percent of current retail sales falls short of the amount raised for schools by property taxes. The critics are correct on this point. Nevertheless, implementing the plan will probably not reduce the money going to Aroca's schools. Several large retailers have selected Aroca City as the site for huge new stores, and these are certain to draw large numbers of shoppers from neighboring municipalities, where sales are taxed at rates of six percent and more. In consequence, retail sales in Aroca City are bound to increase substantially.
Tutor
Joined: 17 Jul 2019
Posts: 1304
Own Kudos [?]: 2286 [1]
Given Kudos: 66
Location: Canada
GMAT 1: 780 Q51 V45
GMAT 2: 780 Q50 V47
GMAT 3: 770 Q50 V45
Send PM
Re: Aroca City currently funds its public schools through taxes on [#permalink]
1
Kudos
Expert Reply
Barfi wrote:
The second statement in bold explains/provides key reason as to why criticism is endorsed.

This is the part you got wrong, Barfi. It’s actually explaining what the critics failed to consider. By the way, a big hint is the word “nevertheless”.

Posted from my mobile device
Intern
Intern
Joined: 07 Oct 2016
Posts: 8
Own Kudos [?]: 1 [0]
Given Kudos: 459
Send PM
Re: Aroca City currently funds its public schools through taxes on [#permalink]
Dear Expert,
Please explain, why option B is wrong?
IIM School Moderator
Joined: 04 Sep 2016
Posts: 1261
Own Kudos [?]: 1238 [0]
Given Kudos: 1207
Location: India
WE:Engineering (Other)
Send PM
Re: Aroca City currently funds its public schools through taxes on [#permalink]
GMATNinjaTwo
GMATNinja VeritasPrepKarishma generis

Quote:
(A) The first is an objection that has been raised against a certain plan; the second is a prediction that, if accurate, undermines the force of that objection.


Is the second part sophisticated way of saying:
The prediction (if accurate) strengthens the objection.
Manager
Manager
Joined: 23 Feb 2012
Posts: 119
Own Kudos [?]: 107 [0]
Given Kudos: 1356
Location: India
Concentration: Strategy, Technology
Send PM
Re: Aroca City currently funds its public schools through taxes on [#permalink]
GMATNinja wrote:
Very nice work, aragonn! I'll throw in my two cents, just in case it's useful at all.

sdlife wrote:
Can you please explain why D is wrong here? I read the entire thread but can't find a convincing answer. I have doubts on both parts of D. For BF1 - seems okay since the argument is refuting that the three percent falls short of the amount raised. I am not sure what's wrong with the second boldface. Please advise. Thanks!

Let's be very clear about what the author is saying:

Quote:
Aroca City currently funds its public schools through taxes on property. In place of this system, the city plans to introduce a sales tax of three percent on all retail sales in the city. Critics protest that three percent of current retail sales falls short of the amount raised for schools by property taxes. The critics are correct on this point. Nevertheless, implementing the plan will probably not reduce the money going to Aroca's schools. Several large retailers have selected Aroca City as the site for huge new stores, and these are certain to draw large numbers of shoppers from neighboring municipalities, where sales are taxed at rates of six percent and more. In consequence, retail sales in Aroca City are bound to increase substantially.

  • BF1 is a claim made by critics.
  • Immediately after BF1, the author writes, "The critics are correct on this point." The author accepts this particular claim without denial or hesitation.

Now lets' be clear about what (D) is saying:
Quote:
The first is a claim that the argument seeks to refute; the second is the main point used by the argument to show that the claim is false.

  • Is the author refuting BF1? Absolutely not. The author says outright that BF1 is correct.
  • Since the first half of this answer choice is wrong, we can eliminate this choice and move on.

I hope this explanation is more than three percent helpful! (Yeah... I'm practicing my dad jokes.)
Hi GmatNinja,
Still D is not clear. I got d point that author outright agreed wd critics. But I think tht 'Argument' as a whole is refuting d first bold face sentence.
Pls correct my approach or understanding.
Thanks.

Sent from my Redmi 5 Plus using GMAT Club Forum mobile app
Manager
Manager
Joined: 09 Apr 2017
Status:Turning my handicaps into assets
Posts: 113
Own Kudos [?]: 44 [0]
Given Kudos: 135
Send PM
Re: Aroca City currently funds its public schools through taxes on [#permalink]
Dear experts,

Please share your views on the following matter.

The reason why I didn't select A was "if accurate": The first is an objection that has been raised against a certain plan; the second is a prediction that, if accurate, undermines the force of that objection.

Aroca City currently funds its public schools through taxes on property. In place of this system, the city plans to introduce a sales tax of three percent on all retail sales in the city. Critics protest that three percent of current retail sales falls short of the amount raised for schools by property taxes. The critics are correct on this point. Nevertheless, implementing the plan will probably not reduce the money going to Aroca's schools. Several large retailers have selected Aroca City as the site for huge new stores, and these are certain to draw large numbers of shoppers from neighboring municipalities, where sales are taxed at rates of six percent and more. In consequence, retail sales in Aroca City are bound to increase substantially.

Doesn't the "if accurate" call the argument into question?Aren't we certain that these facts are indeed true?
Re: Aroca City currently funds its public schools through taxes on [#permalink]
GMATNinja wrote:
Very nice work, aragonn! I'll throw in my two cents, just in case it's useful at all.

sdlife wrote:
Can you please explain why D is wrong here? I read the entire thread but can't find a convincing answer. I have doubts on both parts of D. For BF1 - seems okay since the argument is refuting that the three percent falls short of the amount raised. I am not sure what's wrong with the second boldface. Please advise. Thanks!

Let's be very clear about what the author is saying:

Quote:
Aroca City currently funds its public schools through taxes on property. In place of this system, the city plans to introduce a sales tax of three percent on all retail sales in the city. Critics protest that three percent of current retail sales falls short of the amount raised for schools by property taxes. The critics are correct on this point. Nevertheless, implementing the plan will probably not reduce the money going to Aroca's schools. Several large retailers have selected Aroca City as the site for huge new stores, and these are certain to draw large numbers of shoppers from neighboring municipalities, where sales are taxed at rates of six percent and more. In consequence, retail sales in Aroca City are bound to increase substantially.

  • BF1 is a claim made by critics.
  • Immediately after BF1, the author writes, "The critics are correct on this point." The author accepts this particular claim without denial or hesitation.

Now lets' be clear about what (D) is saying:
Quote:
The first is a claim that the argument seeks to refute; the second is the main point used by the argument to show that the claim is false.

  • Is the author refuting BF1? Absolutely not. The author says outright that BF1 is correct.
  • Since the first half of this answer choice is wrong, we can eliminate this choice and move on.

I hope this explanation is more than three percent helpful! (Yeah... I'm practicing my dad jokes.)

GMATNinja
Sir,
Why the author used the word ''nevertheless''? Did not the author ''refute'' critics' claim by using that word? :? :? :?
Intern
Intern
Joined: 24 Jun 2017
Posts: 27
Own Kudos [?]: 9 [0]
Given Kudos: 70
Send PM
Re: Aroca City currently funds its public schools through taxes on [#permalink]
GMATNinja wrote:
Very nice work, aragonn! I'll throw in my two cents, just in case it's useful at all.

sdlife wrote:
Can you please explain why D is wrong here? I read the entire thread but can't find a convincing answer. I have doubts on both parts of D. For BF1 - seems okay since the argument is refuting that the three percent falls short of the amount raised. I am not sure what's wrong with the second boldface. Please advise. Thanks!

Let's be very clear about what the author is saying:

Quote:
Aroca City currently funds its public schools through taxes on property. In place of this system, the city plans to introduce a sales tax of three percent on all retail sales in the city. Critics protest that three percent of current retail sales falls short of the amount raised for schools by property taxes. The critics are correct on this point. Nevertheless, implementing the plan will probably not reduce the money going to Aroca's schools. Several large retailers have selected Aroca City as the site for huge new stores, and these are certain to draw large numbers of shoppers from neighboring municipalities, where sales are taxed at rates of six percent and more. In consequence, retail sales in Aroca City are bound to increase substantially.

  • BF1 is a claim made by critics.
  • Immediately after BF1, the author writes, "The critics are correct on this point." The author accepts this particular claim without denial or hesitation.

Now lets' be clear about what (D) is saying:
Quote:
The first is a claim that the argument seeks to refute; the second is the main point used by the argument to show that the claim is false.

  • Is the author refuting BF1? Absolutely not. The author says outright that BF1 is correct.
  • Since the first half of this answer choice is wrong, we can eliminate this choice and move on.

I hope this explanation is more than three percent helpful! (Yeah... I'm practicing my dad jokes.)


HI GMAT NInja, On same lines could you explain (B) The first is a criticism, endorsed by the argument, of a funding plan; the second is a point the argument makes in favor of adopting an alternative plan. Isnt the 2nd a point in favour of the alternative plan
GMAT Club Verbal Expert
Joined: 13 Aug 2009
Status: GMAT/GRE/LSAT tutors
Posts: 6920
Own Kudos [?]: 63659 [0]
Given Kudos: 1773
Location: United States (CO)
GMAT 1: 780 Q51 V46
GMAT 2: 800 Q51 V51
GRE 1: Q170 V170

GRE 2: Q170 V170
Send PM
Re: Aroca City currently funds its public schools through taxes on [#permalink]
Expert Reply
srikarkali wrote:
GMATNinja wrote:
Very nice work, aragonn! I'll throw in my two cents, just in case it's useful at all.

sdlife wrote:
Can you please explain why D is wrong here? I read the entire thread but can't find a convincing answer. I have doubts on both parts of D. For BF1 - seems okay since the argument is refuting that the three percent falls short of the amount raised. I am not sure what's wrong with the second boldface. Please advise. Thanks!

Let's be very clear about what the author is saying:

Quote:
Aroca City currently funds its public schools through taxes on property. In place of this system, the city plans to introduce a sales tax of three percent on all retail sales in the city. Critics protest that three percent of current retail sales falls short of the amount raised for schools by property taxes. The critics are correct on this point. Nevertheless, implementing the plan will probably not reduce the money going to Aroca's schools. Several large retailers have selected Aroca City as the site for huge new stores, and these are certain to draw large numbers of shoppers from neighboring municipalities, where sales are taxed at rates of six percent and more. In consequence, retail sales in Aroca City are bound to increase substantially.

  • BF1 is a claim made by critics.
  • Immediately after BF1, the author writes, "The critics are correct on this point." The author accepts this particular claim without denial or hesitation.

Now lets' be clear about what (D) is saying:
Quote:
The first is a claim that the argument seeks to refute; the second is the main point used by the argument to show that the claim is false.

  • Is the author refuting BF1? Absolutely not. The author says outright that BF1 is correct.
  • Since the first half of this answer choice is wrong, we can eliminate this choice and move on.

I hope this explanation is more than three percent helpful! (Yeah... I'm practicing my dad jokes.)


HI GMAT NInja, On same lines could you explain (B) The first is a criticism, endorsed by the argument, of a funding plan; the second is a point the argument makes in favor of adopting an alternative plan. Isnt the 2nd a point in favour of the alternative plan

As nav4042 and VeritasKarishma have pointed out, the issue with (B) is the word "alternative."

Here's the exact language of (B):
Quote:
(B) The first is a criticism, endorsed by the argument, of a funding plan; the second is a point the argument makes in favor of adopting an alternative plan.


The author introduces a plan to raise funds for schools. Let's call this the 3% Plan. The first BF is a criticism of the 3% Plan -- so far, (B) is looking OK. However, the author then goes on to undermine that criticism, and still argue in support of the 3% Plan. He/she doesn't argue for any alternative plan to fund schools (e.g., some other tax or something). Because the author sticks to the 3% Plan, we can't say that the second BF is a point in favor of adopting an alternative plan.

For this reason, (B) is out.

I hope that helps!
Director
Director
Joined: 28 Sep 2018
Posts: 734
Own Kudos [?]: 559 [0]
Given Kudos: 248
GMAT 1: 660 Q48 V33 (Online)
GMAT 2: 700 Q49 V37
Send PM
Re: Aroca City currently funds its public schools through taxes on [#permalink]
GMATNinja VeritasKarishma EducationAisle in (B) and (C) we have "The first is a criticism, endorsed by the argument, of a funding plan" is this correct? Just because the argument mentions The critics are correct on this point can we say that the argument in endorsing this criticism?

Could you elaborate on the word endorsing? Does it mean accepting?
Director
Director
Joined: 28 Sep 2018
Posts: 734
Own Kudos [?]: 559 [0]
Given Kudos: 248
GMAT 1: 660 Q48 V33 (Online)
GMAT 2: 700 Q49 V37
Send PM
Re: Aroca City currently funds its public schools through taxes on [#permalink]
EducationAisle wrote:
Hoozan wrote:
GMATNinja VeritasKarishma EducationAisle in (B) and (C) we have "The first is a criticism, endorsed by the argument, of a funding plan" is this correct? Just because the argument mentions The critics are correct on this point can we say that the argument in endorsing this criticism?

I would say no.

Endorse means express approval. A single line The critics are correct on this point does not mean that the argument endorses this point.

At best, the argument accepts this fact as true but immediately proposes a counter-argument (retail sales in Aroca City are bound to increase substantially) that offsets the point made by the critics.


Keeping the overall argument in mind, and as you mentioned that the author immediately gives a counter-argument, I rejected (B) and (C) also by disagreeing that the argument "endorses" the criticism

But you mentioned that Endorse means approval, so in that case "the critics are correct on this point" seems like an approval... to an extent.
Intern
Intern
Joined: 06 Feb 2021
Posts: 26
Own Kudos [?]: 4 [0]
Given Kudos: 38
GMAT 1: 690 Q50 V34
Send PM
Re: Aroca City currently funds its public schools through taxes on [#permalink]
perezhan wrote:
Aroca City currently funds its public schools through taxes on property. In place of this system, the city plans to introduce a sales tax of three percent on all retail sales in the city. Critics protest that three percent of current retail sales falls short of the amount raised for schools by property taxes. The critics are correct on this point. Nevertheless, implementing the plan will probably not reduce the money going to Aroca's schools. Several large retailers have selected Aroca City as the site for huge new stores, and these are certain to draw large numbers of shoppers from neighboring municipalities, where sales are taxed at rates of six percent and more. In consequence, retail sales in Aroca City are bound to increase substantially.

In the argument given, the two potions in boldface play which of the following roles?

(A) The first is an objection that has been raised against a certain plan; the second is a prediction that, if accurate, undermines the force of that objection.

(B) The first is a criticism, endorsed by the argument, of a funding plan; the second is a point the argument makes in favor of adopting an alternative plan.

(C) The first is a criticism, endorsed by the argument, of a funding plan; the second is the main reason cited by the argument for its endorsement of the criticism.

(D) The first is a claim that the argument seeks to refute; the second is the main point used by the argument to show that the claim is false.

(E) The first is a claim that the argument accepts with certain reservations; the second presents that claim in a rewarding that is not subject to those reservations.


Similar but not the same question from OG is here:
https://gmatclub.com/forum/aroca-city-c ... 37810.html

[quote="VeritasKarishma"][quote="Hoozan"]GMATNinja VeritasKarishma EducationAisle


Can you help me bifurcate each sentence into premise, conclusion etc.
Aroca City currently funds its public schools through taxes on property. In place of this system, the city plans to introduce a sales tax of three percent on all retail sales in the city.-Premise
Critics protest that three percent of current retail sales falls short of the amount raised for schools by property taxes. -Premise
The critics are correct on this point.- Intermediate conclusion as per author
Nevertheless, implementing the plan will probably not reduce the money going to Aroca's schools. - Counter premise
Several large retailers have selected Aroca City as the site for huge new stores, and these are certain to draw large numbers of shoppers from neighboring municipalities, where sales are taxed at rates of six percent and more.-Counter premise
In consequence, [b]retail sales in Aroca City are bound to increase substantially- Main conclusion
Director
Director
Joined: 16 Jun 2021
Posts: 994
Own Kudos [?]: 183 [0]
Given Kudos: 309
Send PM
Re: Aroca City currently funds its public schools through taxes on [#permalink]
perezhan wrote:
Aroca City currently funds its public schools through taxes on property. In place of this system, the city plans to introduce a sales tax of three percent on all retail sales in the city. Critics protest that three percent of current retail sales falls short of the amount raised for schools by property taxes. The critics are correct on this point. Nevertheless, implementing the plan will probably not reduce the money going to Aroca's schools. Several large retailers have selected Aroca City as the site for huge new stores, and these are certain to draw large numbers of shoppers from neighboring municipalities, where sales are taxed at rates of six percent and more. In consequence, retail sales in Aroca City are bound to increase substantially.

In the argument given, the two potions in boldface play which of the following roles?


(A) The first is an objection that has been raised against a certain plan; the second is a prediction that, if accurate, undermines the force of that objection.
Exactly the reasoning and the roles played by the bold face therefore let us hang on to it

(B) The first is a criticism, endorsed by the argument, of a funding plan; the second is a point the argument makes in favor of adopting an alternative plan.
First isn't a criticism but rather a fact that's portrayed and it's not portraying any alternative plans buut rather lending support to the initial plans

(C) The first is a criticism, endorsed by the argument, of a funding plan; the second is the main reason cited by the argument for its endorsement of the criticism.
This is not making any sense since it's way outside the reasoning circle hence out

(D) The first is a claim that the argument seeks to refute; the second is the main point used by the argument to show that the claim is false.
The claim was never false in the first instance but rather it was falling short of the requisite plans hence out

(E) The first is a claim that the argument accepts with certain reservations; the second presents that claim in a rewording that is not subject to those reservations.
These reservations are highly subjective and we are not in a position to credibly comment about the same

Therefore IMO A
Manager
Manager
Joined: 22 Apr 2021
Posts: 131
Own Kudos [?]: 11 [0]
Given Kudos: 409
Send PM
Aroca City currently funds its public schools through taxes on [#permalink]
KarishmaB GMATNinja AjiteshArun DmitryFarber ExpertsGlobal5
mikemcgary

Hi Experts,

I want to confirm whether there is difference between main point and main conclusion. In option(D), "The first is a claim that the argument seeks to refute; the second is the 'main point' used by the argument to show that the claim is false.", we have the word 'main point'.

As per my understanding, Boldface 2 is the main point used by the argument to support that the proposed plan will work. However, Boldface 2 is not the main point of argument. The main point of argument is 'implementing the plan will probably not reduce the money going to Aroca's schools'.

Is my understanding on keyword main point correct?
Intern
Intern
Joined: 17 Oct 2022
Posts: 16
Own Kudos [?]: 3 [0]
Given Kudos: 169
Send PM
Re: Aroca City currently funds its public schools through taxes on [#permalink]
Can someone pls explain why option C is incorrect?

In the argument given, the two potions in boldface play which of the following roles?

(C) The first is a criticism, endorsed by the argument, of a funding plan; the second is the main reason cited by the argument for its endorsement of the criticism.

Can we not argue that the criticism was endorsed due to the below text in bold mentioned:
Critics protest that three percent of current retail sales falls short of the amount raised for schools by property taxes. The critics are correct on this point.

Further, critics stating that 'three percent of current retail sales falls short of the amount raised for schools by property taxes' is indeed a criticism.

Lastly, the second statement in bold explains/provides key reason as to why criticism is endorsed.

Given these factors, I am confused as to why option A> option C?
GMAT Club Bot
Re: Aroca City currently funds its public schools through taxes on [#permalink]
   1   2   3   
Moderators:
GMAT Club Verbal Expert
6920 posts
GMAT Club Verbal Expert
238 posts
CR Forum Moderator
832 posts

Powered by phpBB © phpBB Group | Emoji artwork provided by EmojiOne